{ "cells": [ { "cell_type": "markdown", "metadata": { "tags": [] }, "source": [ "# Fitting a Model to Experimental Data" ] }, { "cell_type": "markdown", "metadata": { "tags": [] }, "source": [ "**Learning Goals**\n", "\n", "* This notebook demonstrates how to 'tuning' a model (i.e., find model parameters) to fit experimental data." ] }, { "cell_type": "markdown", "metadata": { "tags": [] }, "source": [ "## Why do we fit mathematical models to data?\n", "\n", "1. **Test our understanding of a chemical or biological process.** Karl Popper (1902-1994) was a philosopher of science who proposed empirical falsification as the foundation of the scientific method. A hypothesis can never be proven, but it can be falsified by empirical measurement. A mathematical model is a quantitative hypothesis for how a chemical or biological process behaves. Fitting a model to experimental data provides an opportunity to falsify a hypothesis.\n", "\n", "2. **Use the model for process analysis and rational design.**\n", "\n", "3. **Make quantitative predictions of system response.** This is how we will be using models for process control ... that is to use models to solve for inputs that achieve a desired behavior." ] }, { "cell_type": "markdown", "metadata": {}, "source": [ "## Step Test Data\n", "\n", "**Starting at steady state**, record the response of a system to a step change in a manipulable input. In this case, we impose a step input of 50% power to the first heater of the temperature control lab, with P1 = 200." ] }, { "cell_type": "code", "execution_count": 23, "metadata": {}, "outputs": [ { "data": { "text/html": [ "
\n", "\n", "\n", " \n", " \n", " \n", " \n", " \n", " \n", " \n", " \n", " \n", " \n", " \n", " \n", " \n", " \n", " \n", " \n", " \n", " \n", " \n", " \n", " \n", " \n", " \n", " \n", " \n", " \n", " \n", " \n", " \n", " \n", " \n", " \n", " \n", " \n", " \n", " \n", " \n", " \n", " \n", " \n", " \n", " \n", " \n", " \n", " \n", " \n", " \n", " \n", " \n", " \n", " \n", " \n", " \n", "
T1T2Q1Q2
Time
0.021.22121.44650.00.0
10.021.25321.51150.00.0
20.022.18821.57550.00.0
30.023.47721.67250.00.0
40.024.99121.89750.00.0
\n", "
" ], "text/plain": [ " T1 T2 Q1 Q2\n", "Time \n", "0.0 21.221 21.446 50.0 0.0\n", "10.0 21.253 21.511 50.0 0.0\n", "20.0 22.188 21.575 50.0 0.0\n", "30.0 23.477 21.672 50.0 0.0\n", "40.0 24.991 21.897 50.0 0.0" ] }, "metadata": {}, "output_type": "display_data" }, { "data": { "text/html": [ "
\n", "\n", "\n", " \n", " \n", " \n", " \n", " \n", " \n", " \n", " \n", " \n", " \n", " \n", " \n", " \n", " \n", " \n", " \n", " \n", " \n", " \n", " \n", " \n", " \n", " \n", " \n", " \n", " \n", " \n", " \n", " \n", " \n", " \n", " \n", " \n", " \n", " \n", " \n", " \n", " \n", " \n", " \n", " \n", " \n", " \n", " \n", " \n", " \n", " \n", " \n", " \n", " \n", " \n", " \n", " \n", "
T1T2Q1Q2
Time
560.052.80336.72250.00.0
570.052.80336.78650.00.0
580.052.80336.75450.00.0
590.053.02837.01250.00.0
600.052.64236.94750.00.0
\n", "
" ], "text/plain": [ " T1 T2 Q1 Q2\n", "Time \n", "560.0 52.803 36.722 50.0 0.0\n", "570.0 52.803 36.786 50.0 0.0\n", "580.0 52.803 36.754 50.0 0.0\n", "590.0 53.028 37.012 50.0 0.0\n", "600.0 52.642 36.947 50.0 0.0" ] }, "metadata": {}, "output_type": "display_data" }, { "ename": "AttributeError", "evalue": "'numpy.ndarray' object has no attribute 'plot'", "output_type": "error", "traceback": [ "\u001b[0;31m---------------------------------------------------------------------------\u001b[0m", "\u001b[0;31mAttributeError\u001b[0m Traceback (most recent call last)", "Cell \u001b[0;32mIn[23], line 25\u001b[0m\n\u001b[1;32m 23\u001b[0m fig, ax \u001b[38;5;241m=\u001b[39m plt\u001b[38;5;241m.\u001b[39msubplots(\u001b[38;5;241m2\u001b[39m, \u001b[38;5;241m2\u001b[39m, figsize\u001b[38;5;241m=\u001b[39m(\u001b[38;5;241m8\u001b[39m, \u001b[38;5;241m5\u001b[39m))\n\u001b[1;32m 24\u001b[0m ax[\u001b[38;5;241m0\u001b[39m, \u001b[38;5;241m0\u001b[39m]\u001b[38;5;241m.\u001b[39mplot(t, data[\u001b[38;5;124m\"\u001b[39m\u001b[38;5;124mT1\u001b[39m\u001b[38;5;124m\"\u001b[39m], lw\u001b[38;5;241m=\u001b[39m\u001b[38;5;241m2\u001b[39m, label\u001b[38;5;241m=\u001b[39m\u001b[38;5;124m\"\u001b[39m\u001b[38;5;124mT1\u001b[39m\u001b[38;5;124m\"\u001b[39m)\n\u001b[0;32m---> 25\u001b[0m \u001b[43max\u001b[49m\u001b[43m[\u001b[49m\u001b[38;5;241;43m0\u001b[39;49m\u001b[43m]\u001b[49m\u001b[38;5;241;43m.\u001b[39;49m\u001b[43mplot\u001b[49m(t, data[\u001b[38;5;124m\"\u001b[39m\u001b[38;5;124mT2\u001b[39m\u001b[38;5;124m\"\u001b[39m], lw\u001b[38;5;241m=\u001b[39m\u001b[38;5;241m2\u001b[39m, label\u001b[38;5;241m=\u001b[39m\u001b[38;5;124m\"\u001b[39m\u001b[38;5;124mT2\u001b[39m\u001b[38;5;124m\"\u001b[39m)\n\u001b[1;32m 26\u001b[0m ax[\u001b[38;5;241m0\u001b[39m]\u001b[38;5;241m.\u001b[39mset_xlabel(\u001b[38;5;124m\"\u001b[39m\u001b[38;5;124mtime / seconds\u001b[39m\u001b[38;5;124m\"\u001b[39m)\n\u001b[1;32m 27\u001b[0m ax[\u001b[38;5;241m0\u001b[39m]\u001b[38;5;241m.\u001b[39mset_ylabel(\u001b[38;5;124m\"\u001b[39m\u001b[38;5;124mtemperture / °C\u001b[39m\u001b[38;5;124m\"\u001b[39m)\n", "\u001b[0;31mAttributeError\u001b[0m: 'numpy.ndarray' object has no attribute 'plot'" ] }, { "data": { "image/png": "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\n", "text/plain": [ "
" ] }, "metadata": {}, "output_type": "display_data" } ], "source": [ "import numpy as np\n", "import matplotlib.pyplot as plt\n", "import pandas as pd\n", "\n", "# data set\n", "github_repo = \"https://jckantor.github.io/cbe30338-2021/\"\n", "data_file = \"data/Model_Data.csv\"\n", "\n", "# read data set\n", "data = pd.read_csv(github_repo + data_file)\n", "data = data.set_index(\"Time\")[0:]\n", "display(data.head())\n", "display(data.tail())\n", "\n", "# known parameters\n", "T_amb = 21 # deg C\n", "alpha = 0.00016 # watts / (units P1 * percent U1)\n", "P1 = 200 # P1 units\n", "U1 = 50 # steady state value of u1 (percent)\n", "\n", "# visualization\n", "t = data.index\n", "fig, ax = plt.subplots(2, 2, figsize=(8, 5))\n", "ax[0, 0].plot(t, data[\"T1\"], lw=2, label=\"T1\")\n", "ax[0].plot(t, data[\"T2\"], lw=2, label=\"T2\")\n", "ax[0].set_xlabel(\"time / seconds\")\n", "ax[0].set_ylabel(\"temperture / °C\")\n", "ax[0].set_title(f\"Step Response (P1 = {P1}, U1 = {U1})\")\n", "ax[0].legend()\n", "ax[0].grid(True)\n", "\n", "ax[1].plot(t, data[\"Q1\"], lw=2, label=\"Q1\")\n", "ax[1].plot(t, data[\"Q2\"], lw=2, label=\"Q2\")\n", "ax[1].set_xlabel(\"time / seconds\")\n", "ax[1].set_ylabel(\"% of full power\")\n", "ax[1].set_title(f\"Step Response ({P1=}, {U1=})\")\n", "ax[1].set_ylim(-10, 100)\n", "ax[1].legend()\n", "ax[1].grid(True)\n", "\n", "fig.tight_layout()" ] }, { "cell_type": "markdown", "metadata": {}, "source": [ "## Model\n", "\n", "$$\\begin{align}\n", "C_p\\frac{dT_1}{dt} & = U_a(T_{amb} - T_1) + \\alpha P_1u_1\n", "\\end{align}$$" ] }, { "cell_type": "markdown", "metadata": {}, "source": [ "### First Order Linear Differential Equations\n", "\n", "Manipulation will bring this model into a commonly used standard form\n", "\n", "$$\\begin{align}\n", "\\frac{dT_1}{dt} & = \\underbrace{- \\frac{U_a}{C_p}}_a \\underbrace{(T_1 - T_{amb})}_x + \\underbrace{\\frac{\\alpha P_1}{C_p}}_b u_1\n", "\\end{align}$$\n", "\n", "A standard form for a single differential equation is\n", "\n", "$$\\begin{align}\n", "\\frac{dx}{dt} & = ax + bu\n", "\\end{align}$$\n", "\n", "\n", "where $a$ and $b$ are model constants, $x$ is the dependent variable, $x$ is the state variable, and $u$ is the manipulable input. " ] }, { "cell_type": "markdown", "metadata": {}, "source": [ "### Steady State Response\n", "\n", "For a constant value $\\bar{u}$, the steady state response $\\bar{x}$ is given by solution to the equation\n", "\n", "$$\\begin{align}\n", "0 & = a\\bar{x} + b\\bar{u}\n", "\\end{align}$$\n", "\n", "which is\n", "\n", "$$\\begin{align}\n", "\\bar{x} & = -\\frac{b}{a} \\bar{u}\n", "\\end{align}$$" ] }, { "cell_type": "markdown", "metadata": { "tags": [] }, "source": [ "### Transient Response\n", "\n", "The transient response is given by\n", "\n", "$$\\begin{align}\n", "x(t) & = \\bar{x} + \\left[x(t_0) - \\bar{x}\\right] e^{a(t-t_0)}\n", "\\end{align}$$\n", "\n", "which is an exact, analytical solution." ] }, { "cell_type": "markdown", "metadata": {}, "source": [ "#### Apply to Model Equation\n", "\n", "We now apply this textbook solution to the model equation. Comparing equations, we make the following identifications\n", "\n", "$$\n", "\\begin{align}\n", "T_1 - T_{amb} & \\sim x \\\\\n", "-\\frac{U_a}{C_p} & \\sim a \\\\\n", "\\frac{\\alpha P_1}{C_p} & \\sim b \\\\\n", "u_1 & \\sim u\n", "\\end{align}\n", "$$\n", "\n", "Substituting these terms into the standard solution we confirm the steady-state solution found above, and provides a solution for the transient response of the deviation variables.\n", "\n", "$$\\begin{align}\n", "\\bar{x} = -\\frac{b}{a}\\bar{u} \\qquad & \\Rightarrow \\qquad \\bar{T}_{1} = T_{amb} + \\frac{\\alpha P_1}{U_a}\\bar{u}_{1} \\\\\n", "x(t) = \\bar{x} + \\left[x(t_0) - \\bar{x}\\right] e^{a(t-t_0)} \\qquad & \\Rightarrow \\qquad\n", "T_1(t) = T_{amb} + \\frac{\\alpha P_1}{U_a}\\bar{u}_{1} + \\left[T_1(t_0) - T_{amb} - \\frac{\\alpha P_1}{U_a}\\bar{u}_{1}\\right]e^{-\\frac{U_a}{C_p}(t-t_0)}\n", "\\end{align}$$" ] }, { "cell_type": "markdown", "metadata": {}, "source": [ "## Plot a fit\n", "\n", "Let's try a guess fit." ] }, { "cell_type": "code", "execution_count": 32, "metadata": {}, "outputs": [ { "data": { "image/png": "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\n", "text/plain": [ "
" ] }, "metadata": {}, "output_type": "display_data" } ], "source": [ "import numpy as np\n", "import matplotlib.pyplot as plt\n", "import pandas as pd\n", "\n", "# measured data\n", "t = data.index\n", "T1 = data[\"T1\"]\n", "\n", "# known parameters\n", "T_amb = 21 # deg C\n", "alpha = 0.00016 # watts / (units P1 * percent U1)\n", "P1 = 200 # P1 units\n", "U1 = 50 # steady state value of u1 (percent)\n", "\n", "# model p\n", "Ua = 0.05 # watts/deg C\n", "Cp = 9 # joules/deg C\n", "\n", "# model\n", "T1_ss = T_amb + alpha*P1*U1/Ua\n", "T1_model = T1_ss + (T_amb - T1_ss)*np.exp(-Ua*t/Cp)\n", "\n", "# visualization\n", "fig, ax = plt.subplots(1, 1, figsize=(10, 5))\n", "ax = [ax]\n", "ax[0].plot(t, T1, lw=2, label=\"expt\")\n", "ax[0].plot(t, T1_model, '--', lw=2, label=\"model\")\n", "ax[0].legend()\n", "ax[0].set_xlabel(\"time / seconds\")\n", "ax[0].set_ylabel(\"temperture / °C\")\n", "ax[0].set_title(f\"Step Response (P1 = {P1}, U1 = {U1})\")\n", "ax[0].grid(True)" ] }, { "cell_type": "markdown", "metadata": {}, "source": [ "## Improving the fit" ] }, { "cell_type": "code", "execution_count": 15, "metadata": {}, "outputs": [ { "data": { "image/png": "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\n", "text/plain": [ "
" ] }, "metadata": {}, "output_type": "display_data" } ], "source": [ "import numpy as np\n", "import matplotlib.pyplot as plt\n", "import pandas as pd\n", "\n", "# measured data\n", "t = data.index\n", "T1 = data[\"T1\"]\n", "\n", "# known parameters\n", "T_amb = 21 # deg C\n", "alpha = 0.00016 # watts / (units P1 * percent U1)\n", "P1 = 200 # P1 units\n", "U1 = 50 # steady state value of u1 (percent)\n", "\n", "# model p\n", "Ua = 0.05 # watts/deg C\n", "Cp = 6 # joules/deg C\n", "\n", "# model\n", "T1_ss = T_amb + alpha*P1*U1/Ua\n", "T1_model = T1_ss + (T_amb - T1_ss)*np.exp(-Ua*t/Cp)\n", "\n", "# visualization\n", "fig, ax = plt.subplots(1, 1, figsize=(10,5))\n", "ax = [ax]\n", "ax[0].plot(t, T1, lw=2, label=\"expt\")\n", "ax[0].plot(t, T1_model, '--', lw=2, label=\"model\")\n", "ax[0].legend()\n", "ax[0].set_xlabel(\"time / seconds\")\n", "ax[0].set_ylabel(\"temperture / °C\")\n", "ax[0].set_title(f\"Step Response (P1 = {P1}, U1 = {U1})\")\n", "ax[0].grid(True)" ] }, { "cell_type": "markdown", "metadata": {}, "source": [ "How do you handle an initial condition different from $T_{amb}$?" ] }, { "cell_type": "markdown", "metadata": {}, "source": [ "## Measuring the Fit\n", "\n", "* Sum of Squares\n", "\n", "$$SOS = \\sum_n (T^{model}_1(t_k) - T^{expt}_1(t_i))^2$$\n", "\n", "* Sum of Absolute Values\n", "\n", "$$SAE = \\sum_n |T^{model}_1(t_k) - T^{expt}_1(t_i)|$$" ] }, { "cell_type": "code", "execution_count": 43, "metadata": {}, "outputs": [ { "data": { "text/plain": [ "434.7646349068996" ] }, "execution_count": 43, "metadata": {}, "output_type": "execute_result" }, { "data": { "image/png": "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\n", "text/plain": [ "
" ] }, "metadata": {}, "output_type": "display_data" } ], "source": [ "import numpy as np\n", "import matplotlib.pyplot as plt\n", "import pandas as pd\n", "\n", "data_file = \"data/Model_Data.csv\"\n", "data = pd.read_csv(\"https://jckantor.github.io/cbe30338-2021/\" + data_file).set_index(\"Time\")[1:]\n", "t = data.index\n", "T1 = data['T1'].values\n", "\n", "# known parameters\n", "T_amb = 21 # deg C\n", "alpha = 0.00016 # watts / (units P1 * percent U1)\n", "P1 = 200 # P1 units\n", "U1 = 50 # steady state value of u1 (percent)\n", "\n", "# model p\n", "Ua = 0.08 # watts/deg C\n", "Cp = 8 # joules/deg C\n", "\n", "def compare(p, plot=False):\n", " \n", " Ua, Cp = p\n", "\n", " # model\n", " T1_dev_initial = 0\n", " T1_dev_ss = alpha*P1*U1/Ua\n", " T1_dev = T1_dev_ss + (T1_dev_initial - T1_dev_ss)*np.exp(-Ua*t/Cp)\n", " T1_model = T1_dev + T_amb\n", "\n", " # model mismatch\n", " sse = sum((T1_model - T1)**2)\n", " sae = sum(np.abs(T1_model - T1))\n", "\n", " # visualization\n", " if plot:\n", " fig, ax = plt.subplots(4, 1, figsize=(10,10))\n", " ax[0].plot(t, T1, 'r', lw=2)\n", " ax[0].plot(t, T1_model, 'r--', lw=2)\n", " ax[0].axhline(T_amb)\n", " ax[0].axvline(0)\n", " ax[0].set_xlabel(\"time / seconds\")\n", " ax[0].set_ylabel(\"temperture / °C\")\n", " ax[0].set_title(f\"Step Response (P1 = {P1}, U1 = {U1})\")\n", " ax[0].grid(True)\n", "\n", " ax[1].plot(t, T1_model - T1)\n", " ax[1].axhline(0)\n", " ax[1].axvline(0)\n", " ax[1].fill_between(t, T1_model - T1, alpha=0.2)\n", " ax[1].set_title(f'Model Error')\n", " ax[1].set_xlabel(\"time / seconds\")\n", " ax[1].set_ylabel(\"temperture / °C\")\n", " ax[1].grid(True)\n", "\n", " ax[2].plot(t, (T1_model - T1)**2)\n", " ax[2].axhline(0)\n", " ax[2].axvline(0)\n", " ax[2].fill_between(t, (T1_model - T1)**2, alpha=0.2)\n", " ax[2].set_title(f'Sum of Squares = {sse:0.1f}')\n", " ax[2].set_xlabel(\"time / seconds\")\n", " ax[2].set_ylabel(\"temperture / °C\")\n", " ax[2].grid(True)\n", "\n", " ax[3].plot(t, np.abs(T1_model - T1))\n", " ax[3].axhline(0)\n", " ax[3].axvline(0)\n", " ax[3].fill_between(t, np.abs(T1_model - T1), alpha=0.2)\n", " ax[3].set_title(f'Sum of Absolute Values = {sae:0.1f}')\n", " ax[3].set_xlabel(\"time / seconds\")\n", " ax[3].set_ylabel(\"temperture / °C\")\n", " ax[3].grid(True)\n", "\n", " plt.tight_layout()\n", "\n", " return sae \n", " \n", "compare([Ua, Cp], plot=True)" ] }, { "cell_type": "markdown", "metadata": {}, "source": [ "## An Aside: \"The Impact of the Highly Improbable\"\n", "\n", "### Gaussian (or Normal) Distribution\n", "\n", "![](https://sphweb.bumc.bu.edu/otlt/MPH-Modules/PH717-QuantCore/PH717-Module6-RandomError/Normal%20Distribution%20deviations.png)\n", "\n", "### Heavy Tail Distributions\n", "\n", "![](http://www.bitsofscience.org/wordpress-3.0.1/wordpress/wp-content/uploads//extreme-weather-gaussian.jpg)\n", "\n", "![](https://i.stack.imgur.com/OZctU.png)\n", "\n", "### Conclusion: Rare events may not be so rare. Avoid overfitting extreme values!" ] }, { "cell_type": "markdown", "metadata": {}, "source": [ "## Finding a best fit" ] }, { "cell_type": "code", "execution_count": 44, "metadata": {}, "outputs": [ { "name": "stdout", "output_type": "stream", "text": [ "Optimization terminated successfully.\n", " Current function value: 37.283064\n", " Iterations: 48\n", " Function evaluations: 94\n", "[0.04800523 8.72034542]\n" ] }, { "data": { "image/png": "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\n", "text/plain": [ "
" ] }, "metadata": {}, "output_type": "display_data" } ], "source": [ "from scipy.optimize import fmin\n", "\n", "p = fmin(compare, [Ua, Cp])\n", "compare(p, plot=True)\n", "print(p)" ] }, { "cell_type": "code", "execution_count": null, "metadata": {}, "outputs": [], "source": [] }, { "cell_type": "markdown", "metadata": {}, "source": [ "Let's fit a model to the data." ] }, { "cell_type": "code", "execution_count": 42, "metadata": {}, "outputs": [ { "data": { "image/png": "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\n", "text/plain": [ "
" ] }, "metadata": {}, "output_type": "display_data" } ], "source": [ "import numpy as np\n", "import matplotlib.pyplot as plt\n", "import pandas as pd\n", "\n", "data_file = \"data/Model_Data.csv\"\n", "data = pd.read_csv(\"https://jckantor.github.io/cbe30338-2021/\" + data_file).set_index(\"Time\")[1:]\n", "t = data.index\n", "T1 = data['T1'].values\n", "\n", "# parameter values and units\n", "alpha = 0.00016 # watts / (units P1 * percent U1)\n", "P1 = 200 # P1 units\n", "Ua = 0.05 # watts/deg C\n", "Cp = 6 # joules/deg C\n", "U1 = 50 # steady state value of u1 (percent)\n", "T_amb = 21 # deg C\n", "\n", "def compare(Ua, Cp):\n", " T1_dev_initial = 0\n", " T1_dev_ss = alpha*P1*U1/Ua\n", " T1_dev = T1_dev_ss + (T1_dev_initial - T1_dev_ss)*np.exp(-Ua*t/Cp)\n", " T1_model = T1_dev + T_amb\n", " \n", " fig, ax = plt.subplots(2, 1, figsize=(10,8))\n", " \n", " ax[0].plot(t, T1, t, T1_model)\n", " ax[0].set_xlabel('time / seconds')\n", " ax[0].set_ylabel('temperture / °C')\n", " ax[0].grid(True)\n", "\n", " ax[0].text(200, 30, f'Ua = {Ua}')\n", " ax[0].text(200, 25, f'Cp = {Cp}')\n", " \n", " ax[1].plot(t, T1_model - T1)\n", " ax[1].set_title(f'Sum of errors = {sum(abs(T1_model-T1)):0.1f}')\n", " ax[1].grid(True)\n", " \n", " plt.tight_layout()\n", "\n", "compare(0.05, 8)" ] }, { "cell_type": "markdown", "metadata": {}, "source": [ ":::{admonition} Study Question\n", "\n", "1. By trial and error using the `compare()` function defined above, tetermine values for $U_a$ and $C_p$ that yield a good fit of the model to the data. \n", "\n", "1. Are you able to remove all systematic error? If not, why not?\n", "\n", "1. The sum of absolute errors is shown on the chart. Try to find values of $U_a$ and $C_p$ that minimize this error criterion. In your opinion, is that the best choice of model parameters? Why or why not?\n", "\n", "1. Does this solution make sense? The [specific heat capacity for solids](https://en.wikipedia.org/wiki/Heat_capacity) is typically has values in the range of 0.2 to 0.9 watts/degC/gram. Using a value of 0.9 that is typical of aluminum and plastics used for electronic products, what would be the estimated mass of the heater/sensor combination?\n", "\n", "1. Suppose we want to improve the model. Where should we go from here? \n", "\n", ":::" ] }, { "cell_type": "markdown", "metadata": {}, "source": [ "## Exercises\n", "\n", "1. This notebook attempted to fit a first-order model of a heater/sensor assembly to experimental data. In the course of the fit, estimates were derived for parameters $\\alpha$, P1, $U_a$, and $C_p$. From these parameter values, estimate a time constant. \n", "\n", "2. Apply the techniques outlined in Section 2.2 for the estimation of time constants from experimental data. How does that value compare to the calculation in the first question?\n", "\n", "3. It's not always possible to wait for a system to reach steady state. Create a new function, `compare3`, that introduces a third unknown parameter representing an initial condition different for $T'_1(0)$ that is different from zero." ] }, { "cell_type": "code", "execution_count": null, "metadata": {}, "outputs": [], "source": [] } ], "metadata": { "kernelspec": { "display_name": "Python 3 (ipykernel)", "language": "python", "name": "python3" }, "language_info": { "codemirror_mode": { "name": "ipython", "version": 3 }, "file_extension": ".py", "mimetype": "text/x-python", "name": "python", "nbconvert_exporter": "python", "pygments_lexer": "ipython3", "version": "3.9.15" } }, "nbformat": 4, "nbformat_minor": 4 }